LSAT and Law School Admissions Forum

Get expert LSAT preparation and law school admissions advice from PowerScore Test Preparation.

User avatar
 Dave Killoran
PowerScore Staff
  • PowerScore Staff
  • Posts: 5853
  • Joined: Mar 25, 2011
|
#44063
Complete Question Explanation
(The complete setup for this game can be found here: lsat/viewtopic.php?t=14120)

The correct answer choice is (C)

This is a perfect question to attack with the inferences. Think about a variable that, when placed in I, would force another variable into F:

  • ..... ..... ..... ..... ..... HI :arrow: ZI + WF
This means that if H is in I, then W must be in F. So answer choice (C) is correct.
 ranbirdhaliwal11@gmail.com
  • Posts: 1
  • Joined: Feb 10, 2023
|
#103300
Why is E wrong?
 Adam Tyson
PowerScore Staff
  • PowerScore Staff
  • Posts: 5153
  • Joined: Apr 14, 2011
|
#103312
E is incorrect, rainbird, because M and W CAN both serve on the Incentives committee. Here's what that solution would look like:

F: HUZ

I: GLMW

This does not conflict with any of the rules, so it's possible, and this question for asking for something that is not possible

Get the most out of your LSAT Prep Plus subscription.

Analyze and track your performance with our Testing and Analytics Package.